5
$\begingroup$

Can the Lifting The Exponent Lemma ( found at https://web.archive.org/web/20141023141031/http://www.artofproblemsolving.com/Resources/Papers/LTE.pdf) be proven with Hensel's Lemma? How?

0 Answers 0